0% found this document useful (0 votes)
352 views49 pages

2021 AMC 10A Solution

美国数学竞赛AMC10的2021年A卷题解,含多种解法和讲解

Uploaded by

panda.yh0207
Copyright
© © All Rights Reserved
We take content rights seriously. If you suspect this is your content, claim it here.
Available Formats
Download as PDF, TXT or read online on Scribd
0% found this document useful (0 votes)
352 views49 pages

2021 AMC 10A Solution

美国数学竞赛AMC10的2021年A卷题解,含多种解法和讲解

Uploaded by

panda.yh0207
Copyright
© © All Rights Reserved
We take content rights seriously. If you suspect this is your content, claim it here.
Available Formats
Download as PDF, TXT or read online on Scribd
You are on page 1/ 49

2021 AMC 10A Solution

Problem1

What is the value of

Solution 1

This

corresponds to

Solution 2

Problem2
Portia's high school has times as many students as Lara's high school. The two
high schools have a total of students. How many students does Portia's
high school have?

Solution 1
The following system of equations can be formed with representing the
number of students in Portia's high school and representing the number of

students in Lara's high school.


Substituting with we get . Solving for , we get .

Since we need to find we multiply by 3 to get , which

is
Solution 2 (One Variable)
Suppose Lara's high school has students. It follows that Portia's high school

has students. We know that or Our

answer is

Solution 3 (Arithmetics)
Clearly, students is times as many students as Lara's high school.
Therefore, Lara's high school has students, and Portia's

high school has students.

Solution 4 (Answer Choices)

Solution 4.1 (Quick Inspection)


The number of students in Portia's high school must be a multiple of This

eliminates and . Since is too small

(as is clearly true), we are left

with

Solution 4.2 (Plug in the Answer Choices)

For we have So, is incorrect.

For we have So, is


incorrect.

For we have So, is

correct. For completeness, we will check choices and


For we have So, is
incorrect.

For we have So, is


incorrect.

Problem3

The sum of two natural numbers is . One of the two numbers is divisible
by . If the units digit of that number is erased, the other number is obtained.
What is the difference of these two numbers?

Solution 1
The units digit of a multiple of will always be . We add a whenever we
multiply by . So, removing the units digit is equal to dividing by .
Let the smaller number (the one we get after removing the units digit) be . This
means the bigger number would be .

We know the sum is so .


So . The difference is . So, the answer

is .

Solution 2 (Lazy Speed)


Since the ones place of a multiple of is , this implies the other integer has to
end with a since both integers sum up to a number that ends with a . Thus,
the ones place of the difference has to be , and the only answer

choice that ends with an is


~CoolJupiter 2021
Another quick solution is to realize that the sum is represents a number added

to . The difference is , which is of the given sum.


Solution 3 (Vertical Addition and Logic)

Let the larger number be It follows that the smaller number

is Adding vertically, we have


Working from right to left, we
have The larger number

is and the smaller number is Their difference

is

Problem4
A cart rolls down a hill, travelling inches the first second and accelerating so
that during each successive -second time interval, it travels inches more than
during the previous -second interval. The cart takes seconds to reach the
bottom of the hill. How far, in inches, does it travel?

Solution 1 (Arithmetic Series)

Since we seek the


sum

in which there are addends. The last addend

is Therefore, the requested sum


is

Recall that to find the sum of an arithmetic series, we take the average of the first
and last terms, then multiply by the number of terms,

namely
Solution 2 (Answer Choices and Modular Arithmetic)

From the -term sum in the previous solution,


taking
modulo gives

The only answer choices that are are and By a

quick estimation, is too small, leaving us with

Problem5
The quiz scores of a class with students have a mean of . The mean
of a collection of of these quiz scores is . What is the mean of the
remaining quiz scores of terms of ?

Solution 1 (Generalized)
The total score in the class is The total score on the quizzes
is Therefore, for the remaining quizzes ( of them),

the total score is Their mean score is

~MRENTHUSIASM

Solution 2 (Convenient Values and Observations)


Set The answer is the same as the last student's quiz score, which
is From the answer choices,

only yields a negative value for

Problem6
Chantal and Jean start hiking from a trailhead toward a fire tower. Jean is
wearing a heavy backpack and walks slower. Chantal starts walking at miles
per hour. Halfway to the tower, the trail becomes really steep, and Chantal slows
down to miles per hour. After reaching the tower, she immediately turns around
and descends the steep part of the trail at miles per hour. She meets Jean at
the halfway point. What was Jean's average speed, in miles per hour, until they
meet?

Solution 1 (Generalized Distance)


Let miles be the distance from the start to the fire tower. When Chantal
meets Jean, she has traveled
for

hours. Jean also has traveled for hours, and he travels for miles. So, his

average speed is miles per hour.

Solution 2 (Convenient Distance)


We use the same template as shown in Solution 1, except that we
replace with a concrete number.
Let miles be the distance from the start to the fire tower. When Chantal

meets Jean, she travels for


hours. Jean also has traveled for hours, and he travels for miles. So, his

average speed is miles per hour.

Problem7
Tom has a collection of snakes, of which are purple and of which are
happy. He observes that all of his happy snakes can add, none of his purple
snakes can subtract, and all of his snakes that can't subtract also can't add.
Which of these conclusions can be drawn about Tom's snakes?

Purple snakes can add.

Purple snakes are happy.

Snakes that can add are purple.

Happy snakes are not purple.

Happy snakes can't subtract.

Solution 1
We know that purple snakes cannot subtract, thus they cannot add either. Since
happy snakes must be able to add, the purple snakes cannot be happy.
Therefore, we know that the happy snakes are not purple and the answer

is .

Solution 2 (Explains Solution 1 Using Arrows)


We are given that

Combining and into below, we have

Clearly, the answer is


Problem8
When a student multiplied the number by the repeating

decimal where and are digits, he did not notice the

notation and just multiplied times . Later he found that his answer

is less than the correct answer. What is the -digit number

Solution

It is known that and . Let . We have

that . Solving gives

that so .

Problem9

What is the least possible value of for real


numbers and ?

Solution 1
Expanding, we get that the expression

is or

. By the trivial inequality(all squares are nonnegative)

the minimum value for this is , which can be achieved at .


~aop2014
Solution 2 (Beyond Overkill)
Like solution 1, expand and simplify the original equation

to and

let . To find local extrema, find

where . First, find the first partial derivative with respect to x


and y and find where they

are :

Thus, there is a local extreme at . Because this is the only extreme, we


can assume that this is a minimum because the problem asks for the minimum
(though this can also be proven using the partial second derivative test) and the

global minimum since it's the only minimum, meaning is the minimum

of . Plugging into , we find 1

Problem10
Which of the following is equivalent
to

Solution 1

All you need to do is multiply the entire equation by . Then all the terms
will easily simplify by difference of squares and you will

get or as your final answer. Notice you don't need to worry


about because that's equal to .
-Lemonie
Solution 2

If you weren't able to come up with the insight, then you could just

notice that the answer is divisible by ,

and . We can then use Fermat's Little Theorem

for on the answer choices to determine which of the answer

choices are divisible by both and . This is .

Solution 3
After expanding the first few terms, the result after each term appears to
be

where n is the number of terms expanded. We can prove this using


mathematical induction. The base step is trivial. When expanding another term,

all of the previous terms multiplied by would


give

, and all the previous terms multiplied by would


give

. Their sum is equal


to

, so the proof is complete. Since is equal


to

, the answer is .
-SmileKat32

Solution 4 (Engineer's Induction)


We can compute some of the first few partial products, and notice

that . As we don't have to prove

this, we get the product is , and smugly

click . ~rocketsri

Problem11
For which of the following integers is the base-

number not divisible by ?

Solution
We
have

This expression is divisible by unless The only choice

congruent to modulo is

Problem12
Two right circular cones with vertices facing down as shown in the figure below
contains the same amount of liquid. The radii of the tops of the liquid surfaces
are cm and cm. Into each cone is dropped a spherical marble of radius cm,
which sinks to the bottom and is completely submerged without spilling any liquid.
What is the ratio of the rise of the liquid level in the narrow cone to the rise of the
liquid level in the wide cone?
Solution 1 (Use Tables to Organize Information)
Initial Scenario

By
similar triangles:

For the narrow cone, the ratio of base radius to height is which remains
constant.

For the wide cone, the ratio of base radius to height is which remains
constant.

Equating the initial volumes gives which simplifies

to
Final Scenario (Two solutions follow from here.)
Solution 1.1 (Fraction Trick)
Let the base radii of the narrow cone and the wide cone

be and respectively, where We have the following


table:

Equating the final volumes gives which simplifies

to or

Lastly, the requested ratio

is
PS:
1. This problem uses the following fraction trick:
For unequal positive

numbers and if then

Quick Proof

From we know that and .

Therefore,
2. The work above shows that, regardless of the shape or the volume of the solid
dropped in, as long as the solid sinks to the bottom and is completely submerged
without spilling any liquid, the answer will remain unchanged.
~MRENTHUSIASM
Solution 1.2 (Bash)
Let the base radii of the narrow cone and the wide cone
be and respectively.
Let the rises of the liquid levels of the narrow cone and the wide cone

be and respectively. We have the following


table:

By similar triangles discussed above, we

have

The volume of the marble dropped in is

Now, we set up an equation for the volume of the narrow cone and solve

for
Next, we set up an equation for the volume of the wide cone

Using the exact same process


from above (but with different numbers), we

get Recall that Therefore, the

requested ratio is

~MRENTHUSIASM

Solution 2 (Quick and dirty)


The heights of the cones are not given, so suppose the heights are very large
(i.e. tending towards infinity) in order to approximate the cones as cylinders with
base radii 3 and 6 and infinitely large height. Then the base area of the wide
cylinder is 4 times that of the narrow cylinder. Since we are dropping a ball of the
same volume into each cylinder, the water level in the narrow cone/cylinder

should rise times as much.


Problem13
What is the volume of tetrahedron with edge

lengths , , , ,

, and ?

Solution
Drawing the tetrahedron out and testing side lengths, we realize that the triangles
ABD and ABC are right triangles. It is now easy to calculate the volume of the

tetrahedron using the formula for the volume of a pyramid: , so

we have an answer of .

Problem14
All the roots of the

polynomial are
positive integers, possibly repeated. What is the value of ?

Solution 1:
By Vieta's formulae, the sum of the 6 roots is 10 and the product of the 6 roots is
16. By inspection, we see the roots are 1, 1, 2, 2, 2, and 2, so the function
is

. Therefore, . ~JHawk0224

Solution 2:
Using the same method as Solution 1, we find that the roots

are and . Note that is the negation of the 3rd symmetric sum
of the roots. Using casework on the number of 1's in each of

the products we
obtain

Problem15

Values for and are to be selected

from without replacement (i.e. no two letters have the


same value). How many ways are there to make such choices so that the two

curves and intersect? (The order in


which the curves are listed does not matter; for example, the

choices is considered the same as the

choices )

Solution 1 (Intuition):
Visualizing the two curves, we realize they are both parabolas with the same axis
of symmetry. Now assume that the first equation is above the second, since order
doesn't matter. Then and . Therefore the number of ways to

choose the four integers is , and the answer is . ~IceWolf10

Solution 2 (Algebra):

Setting , we find

that ,

so by the trivial inequality. This implies


that and must both be positive or negative. If two distinct
values are chosen for and respectively, there are ways to
order them so that both the numerator and denominator are positive/negative
(increasing and decreasing). We must divide by at the end, however, since
the curves aren't considered distinct. Calculating, we

get

Problem16
In the following list of numbers, the integer appears times in the list

for .

What is the
median of the numbers in this list?

Solution 1
There

are number
s in total. Let the median be . We want to find the median such

that or Note

that . Plugging this value in

as gives ,
so is the nd and rd numbers, and hence, our desired

answer. .

Note that we can derive through the

formula where is a perfect


square less than or equal to . We set to , so ,

and . We then have .

Solution 2

The th number of this sequence is via the quadratic


formula. We can see that if we halve we end up

getting . This is approximately the number divided

by . and since looks like the only number close to it,

it is answer ~Lopkiloinm

Solution 3 (answer choices)


We can look at answer choice , which is first. That means that the
number of numbers from to is roughly the number of numbers
from to .

The number of numbers from to is which is


approximately The number of numbers

from to is which is
approximately as well. Therefore, we can be relatively sure the answer

choice is
Problem17

Trapezoid has ,

and . Let be the intersection of the

diagonals and , and let be the midpoint of . Given

that , the length of can be written in the form ,


where and are positive integers and is not divisible by the square of any
prime. What is ?

Diagram

~MRENTHUSIASM (by Geometry Expressions)

Solution 1
Angle chasing reveals that ,

therefore Additional angle


chasing shows that ,
therefore
Since is right, the Pythagorean theorem implies

that

Solution 2 (One Pair of Similar Triangles, then Areas)

Since is isosceles with legs and it follows that the

median is also an altitude

of Let and We have

Since by AA, we

have

Let the brackets denote areas. Notice that (By the

same base/height, Subtracting from both

sides gives ). Doubling both sides, we

have

In we have

and Finally,
Solution 3 (short)

Let and is perpendicular bisector

of Let so

(1) so we get or

(2) pythag on gives

(3) with ratio so

Thus, or And

so and the answer

is

Solution 4 - Extending the line


Observe that is congruent to ; both are similar

to . Let's extend and past points and respectively,


such that they intersect at a point . Observe that is degrees,
and
that
.
Thus, by ASA, we know that , thus, ,

meaning is the midpoint of . Let be the midpoint of . Note


that is congruent to , thus ,

meaning is the midpoint of

Therefore, and are both medians of . This means


that is the centroid of ; therefore, because the centroid divides the

median in a 2:1 ratio, . Recall that is the midpoint


of ; . The question tells us
that ; ; we can write this in terms

of ; .

We are almost finished. Each side length of is twice as long as the


corresponding side length or , since those triangles are
similar; this means that . Now, by Pythagorean
theorem
on ,

Problem18

Let be a function defined on the set of positive rational numbers with the

property that for all positive rational

numbers and . Furthermore, suppose that also has the property

that for every prime number . For which of the following

numbers is ?

Solution 1

Looking through the solutions we can see that can be expressed

as so using the prime numbers to


piece together what we have we can get ,

so or .

Solution 2

We know that . By transitive, we

have Subtracting from both sides

gives Also

In we have .

In we have .

In we have .

In we

have .
In we have .

Thus, our answer is

~JHawk0224 ~awesomediabrine

Solution 3 (Deeper)

Consider the rational , for integers. We

have .

So . Let be a prime. Notice

that . And . So

if , .

We simply need this to be greater than what we have for . Notice that for

answer choices and , the numerator has less prime factors


than the denominator, and so they are less likely to work. We check first, and

it works, therefore the answer is .


~yofro

Solution 4 (Most Comprehensive, Similar to Solution 3)


We have the following important results:

for all positive rational numbers and


positive integers
for all positive rational numbers and positive
integers

for all positive rational numbers


Proofs

Result can be shown by induction.

Result Since positive powers are just repeated multiplication of the base,

we will use result to prove result

Result For all positive rational numbers we

have Therefore, we

get So, result is true.

Result For all positive rational numbers we

have It follows

that and result is true.


For all positive integers and suppose and are their
prime factorizations, respectively, we

have

We apply function on each fraction in the choices:

Therefore, the answer is


Solution 5
The problem gives us that f(p)=p. If we let a=p and b=1, we get f(p)=f(p)+f(1),
which implies f(1)=0. Notice that the answer choices are all fractions, which
means we will have to multiply an integer by a fraction to be able to solve it.
Therefore, let's try plugging in fractions and try to solve them. Note that if we plug
in a=p and b=1/p, we get f(1)=f(p)+f(1/p). We can solve for f(1/p) as -f(p)! This
gives us the information we need to solve the problem. Testing out the answer
choices gives us the answer of E.

Problem19
The area of the region bounded by the graph

of is , where and are


integers. What is ?

Solution 1
In order to attack this problem, we need to consider casework:

Case 1:

Substituting and simplifying, we have ,

i.e. , which gives us a circle of radius centered

at .

Case 2:

Substituting and simplifying again, we have ,

i.e. . This gives us a circle of radius centered

at .

Case 3:
Doing the same process as before, we have ,

i.e. . This gives us a circle of radius centered

at .

Case 4:

One last time: we have ,

i.e. . This gives us a circle of radius centered

at .

After combining all the cases and drawing them on the Cartesian Plane, this is
what the diagram looks like:

Now, the area of the


shaded region is just a square with side length with four semicircles of

radius . The area is . The answer

is which is
Problem20

In how many ways can the sequence be rearranged so that no


three consecutive terms are increasing and no three consecutive terms are
decreasing?

Solution 1 (Bashing)
We write out the cases. These cases are the ones that
work:

We count these out and get permutations that work.

Solution 2 (Casework)
Reading the terms from left to right, we have two cases:

( stands for increase and stands for decrease.)

For note that for the second and fourth terms, one of which must be

a and the other one must be a or We have four sub-cases:


For the first two blanks must be and in some order, and the last blank

must be a for a total of possibilities. Similarly, also has possibilities.

For there are no restrictions for the numbers and So, we

have possibilities. Similarly, also has possibilities.

Together, has possibilities. By

symmetry, also has possibilities. Together, the answer

is

Solution 3 (similar to solution 2)


Like Solution 2, we have two cases. Due to symmetry, we just need to count one

of the cases. For the purpose of this solution, we will be doing .


Instead of starting with 5, we start with 1.
There are two ways to place it:
_1_ _ _
_ _ _1_
Now we place 2, it can either be next to 1 and on the outside, or is place in where
1 would go in the other case. So now we have another two "sub case":
_1_2_(case 1)
21_ _ _(case 2)
There are 3! ways to arrange the rest for case 1, since there is no restriction.
For case 2, we need to consider how many ways to arrange 3,4,5 in a a>b<c
fashion. It should seem pretty obvious that b has to be 3, so there will be 2! way
to put 4 and 5.
Now we find our result, times 2 for symmetry, times 2 for placement of 1 and
times (3!+2!) for the two different cases for placement of 2. This give

us .
Solution 4: Symmetry
We only need to find the # of rearrangements when 5 is the 4th digit and 5th digit.
Find the total, and multiply by 2. Then we can get the answer by adding the case
when 5 is the third digit.

Case : 5 is the 5th digit. __ __ __ __ 5


Then can only be either 1st digit or the 3rd digit.
4 __ __ __ 5, then the only way is that is the 3rd digit, so it can be
either or , give us results.
__ __ 4 __ 5, then the 1st digit must be or , gives us way, and gives
us ways. (Can't be because the first digit would increasing). Therefore, in
the middle and in the last would result in ways.

Case : is the fourth digit. __ __ __ 5 __


Then the last digit can be all of the 4 numbers , , , and . Let's say if the last
digit is , then the 2nd digit would be the largest for the remaining digits to
prevent increasing order or decreasing order. Then the remaining two are
interchangeable, give us ways. All of the can work, so case would result

in ways.

Case : is in the middle. __ __ 5 __ __


Then there are only two cases: 1. , then 4 and 3 are interchangeable,
which results in . Or it can be , then 4 and 2 are
interchangeable, but it can not be , so there can only be 2 possible
ways: , .

Therefore, case 3 would result in ways.

, so the total ways for case 1 and case 2 with both


increasing and decreasing would be

Problem21
Let be an equiangular hexagon. The

lines and determine a triangle with area , and the


lines and determine a triangle with area . The

perimeter of hexagon can be expressed as ,


where and are positive integers and is not divisible by the square of

any prime. What is ?

Solution (Misplaced problem?)


Note that the extensions of the given lines will determine an equilateral triangle

because the hexagon is equiangular. The area of the first triangle is ,

so the side length is . The area of the second triangle

is , so the side length is . We can set the first

value equal to and the second equal

to by substituting some lengths in with different sides


of the same equilateral triangle. The perimeter of the hexagon is just the sum of

these two, which is and

Problem22
Hiram's algebra notes are pages long and are printed on sheets of paper;
the first sheet contains pages and , the second sheet contains
pages and , and so on. One day he leaves his notes on the table before
leaving for lunch, and his roommate decides to borrow some pages from the
middle of the notes. When Hiram comes back, he discovers that his roommate
has taken a consecutive set of sheets from the notes and that the average
(mean) of the page numbers on all remaining sheets is exactly . How many
sheets were borrowed?
Solution
Suppose the roommate took pages through , or equivalently, page

numbers through . Because there are numbers


taken,

The first possible solution that comes to mind is


if

, which indeed works, giving and . The answer

is

Solution 2 (Different Variable Choice, Similar Logic)

Suppose the smallest page number removed is and pages are removed. It

follows that the largest page number removed is

Remarks:

1. pages are removed means that sheets are removed, from which must
be even.
2. must be odd, as the smallest page number removed is on the right side
(odd-numbered).

3.

4. The sum of the page numbers removed is


Together, we have
The factors

of are
Since is even, we only have a few cases to consider:

Since only are possible:

If then the note pages will run out if we take pages starting from
page

If then the average page number of the remaining pages will be


undefined, as there is no page remaining (after taking pages starting from
page ).

So, the only possibility is from which pages are taken out,

which is sheets.
Solution 3
Let be the number of sheets borrowed, with an average page

number . The remaining sheets have an average page


number of which is less than , the average page number of all
pages, therefore . Since the borrowed sheets start with an odd page
number and end with an even page number we have . We notice

that and .

The weighted increase of average page number

from to should be equal to the weighted decrease of average


page number from to , where the weights are the page number in each
group (borrowed vs. remained), therefore

Since we have either or .


If then . If then which is

impossible. Therefore the answer should be

Problem23
Frieda the frog begins a sequence of hops on a grid of squares, moving
one square on each hop and choosing at random the direction of each hop-up,
down, left, or right. She does not hop diagonally. When the direction of a hop
would take Frieda off the grid, she "wraps around" and jumps to the opposite
edge. For example if Frieda begins in the center square and makes two hops
"up", the first hop would place her in the top row middle square, and the second
hop would cause Frieda to jump to the opposite edge, landing in the bottom row
middle square. Suppose Frieda starts from the center square, makes at most four
hops at random, and stops hopping if she lands on a corner square. What is the
probability that she reaches a corner square on one of the four hops?
Solution 1 (complementary counting)

We will use complementary counting. First, the frog can go left with probability .
We observe symmetry, so our final answer will be multiplied by 4 for the 4

directions, and since , we will ignore the leading probability.

From the left, she either goes left to another edge ( ) or back to the center ( ).
Time for some casework.
She goes back to the center.
Now, she can go in any 4 directions, and then has 2 options from that edge. This

gives . --End case 1

She goes to another edge (rightmost).

Subcase 1: She goes back to the left edge. She now has 2 places to go, giving

Subcase 2: She goes to the center. Now any move works.

for this case. --End case 2

She goes back to the center in Case 1 with probability , and to the right edge

with probability

So, our answer is

But, don't forget complementary counting. So, we

get . ~ firebolt360

Video Solution for those who prefer: https://youtu.be/ude2rzO1cmk ~ firebolt360


Solution 2 (direct counting and probability states)
We can draw a state diagram with three states: center, edge, and corner. Denote
center by M, edge by E, and corner by C. There are a few ways Frieda can reach
a corner in four or less moves: EC, EEC, EEEC, EMEC. Then, calculating the
probabilities of each of these cases happening, we

have , so

the answer is . ~IceWolf10

Solution 3 (Similar to Solution 2, but Finds the


Numerator and Denominator Separately)
Denominator

There are ways to make hops without restrictions.

Numerator (Casework)
Suppose Frieda makes hops without stopping. We perform casework on which
hop reaches a corner for the first time.

Hop (Hops and have no restrictions)

The independent hops have options, respectively. So, this case


has ways.

Hop (Hop has no restriction)

No matter which direction the first hop takes, the second hop must "wrap
around".

The independent hops have options, respectively. So, this case


has ways.

Hop

Two sub-cases:
The second hop "wraps around". It follows that the third hop also "wraps
around".

The independent hops have options, respectively. So, this sub-


case has ways.

The second hop backs to the center.

The independent hops have options, respectively. So, this sub-


case has ways.

Together, Case has ways.

The numerator is

Probability

Solution 4

Let be the probability that Frieda is on the central square after n

moves, be the probability that Frieda is on one of the four squares on the

middle of the edges after n moves, and (V for vertex) be the probability that
Frieda is on a corner after n moves. The only way to reach the center is by
moving in specific direction out of total directions from the middle of an edge,

so . The ways to reach the middle of an edge are by moving in


any direction from the center or by moving in specific direction from the middle

of an edge, so . The ways to reach a corner are by


simply staying there after reaching there in a previous move or by moving
in specific directions from the middle of an edge, so .

Since Frieda always start from the center, , , and .

We use the previous formulas to work out and find it to be .

Solution 5
Imagine an infinite grid of by squares such that there is a by square

centered at for all ordered pairs of integers

It is easy to see that the problem is equivalent to Frieda moving left, right, up, or

down on this infinite grid starting at . (minus the teleportations) Since


counting the complement set is easier, we'll count the number of -step paths
such that Frieda never reaches a corner point.
In other words, since the reachable corner points

are and Frieda can


only travel along the collection of points included in , where is all points

on and such that and , respectively, plus


all points on the big square with side length centered at We then can
proceed with casework:

Case : Frieda never reaches nor

When Frieda only moves horizontally or vertically for her four moves, she can do

so in ways for each case . Thus, total paths for the


subcase of staying in one direction. (For instance, all length combinations
of and except , , , and for the
horizontal direction.)
There is another subcase where she changes directions during her path. There
are four symmetric cases for this subcase depending on which of the four
quadrants Frieda hugs. For the first quadrant, the possible paths
are , , , and Thus, a total
of ways for this subcase.

Total for Case :

Case : Frieda reaches or .

Once Frieda reaches one of the points listed above (by using three moves), she
has four choices for her last move. Thus, a total of paths for this
case.
Our total number of paths never reaching coroners is

thus making for an answer of

Solution 6 (Casework)
We take cases on the number of hops needed to reach a corner. For simplicity,
denote as a move that takes Frieda to an edge, as wrap-around move
and as a corner move. Also, denote as a move that takes us to the center.

2 Hops

Then, Frieda will have to as her set of moves. There are ways to
move to an edge, and corners to move to, for a total of cases
here. Then, there are choices for each move, for a probability

of .

3 Hops
In this case, Frieda must wrap-around. There's only one possible combination,

just . There are ways to move to an edge, way to wrap-around


(you must continue in the same direction) and corners, for a total
of cases here. Then, there are choices for each move, for a

probability of .

4 Hops
Lastly, there are two cases we must consider here. The first case

is , and the second is . For the first case,


there are ways to move to an edge, way to return to the center, ways to
move to an edge once again, and ways to move to a corner. Hence, there is a
total of cases here. Then, for the second case, there
are ways to move to a corner, way to wrap-around, way to wrap-around
again, and ways to move to a corner. This implies there
are cases here. Then, there is a total

of cases, out of a total of cases, for a probability

of .

Then, the total probability that Frieda ends up on a corner

is , corresponding to choice . ~rocketsri

Solution 7
I denote 3x3 grid by
- HOME square (x1)
- CORN squares (x4)
- SIDE squares (x4)
Transitions:
- HOME always move to SIDE
- CORN is DONE

- SIDE move to CORN with move to SIDE with and

move to CORN with

After one move, will be on square

After two moves, will be

After three moves, will


be

After four moves, probability on CORN will


be

Problem24
The interior of a quadrilateral is bounded by the graphs

of and , where a positive real


number. What is the area of this region in terms of , valid for all ?

Diagram
Graph in Desmos: https://www.desmos.com/calculator/nagjmnkywx
~MRENTHUSIASM

Solution 1
The

conditions and give

and or and
. The slopes here are perpendicular, so the quadrilateral is a
rectangle. Plug in and graph it. We quickly see that the area

is , so the answer can't be or by testing the values they


give (test it!). Now plug in . We see using a ruler that the sides of the

rectangle are about and . So the area is about .

Testing we get which is clearly less than , so it is out. Testing we

get which is near our answer, so we leave it. Testing we get , way

less than , so it is out. So, the only plausible answer is ~firebolt360

Solution 2 (Casework)

For the equation the cases are

This is a line with -intercept -intercept and

slope

This is a line with -intercept -

intercept and slope

For the equation the cases are

This is a line with -intercept -intercept and


slope

This is a line with -intercept -


intercept and slope
Plugging into the choices gives
Plugging into the four above equations and solving systems of equations

for intersecting lines [

and and and and ], we get the respective

solutions

Solution 2.1 (Rectangle)


Since the slopes of the intersecting lines (from the four above equations) are
negative reciprocals, the quadrilateral is a rectangle. Finally, by the Distance

Formula, the length and width of the rectangle are and The area

we seek is

The answer is

~MRENTHUSIASM

Solution 2.2 (Shoelace Formula)


Even if we do not recognize that the solutions form the vertices of a rectangle, we
can apply the Shoelace Formula

on consecutive vertices
The area formula
is

Therefore, the answer is

Suggested Reading for the Shoelace


Formula: https://artofproblemsolving.com/wiki/index.php/Shoelace_Theorem
~MRENTHUSIASM

Solution 3 (Geometry)
Similar to Solution 2, we will use the equations of the four cases:

(1) This is a line with -intercept , -intercept , and

slope

(2) This is a line with -intercept , -intercept ,

and slope

(3)* This is a line with -intercept , -intercept , and


slope
(4)* This is a line with -intercept , -intercept , and
slope
The area of the rectangle created by the four equations can be written
as
=
=

(Note: slope )
-fnothing4994

Solution 4 (bruh moment solution)

Trying narrows down the choices to options , and .

Trying and eliminates and , to

obtain

Problem25
How many ways are there to place indistinguishable red
chips, indistinguishable blue chips, and indistinguishable green chips in the
squares of a grid so that no two chips of the same color are directly
adjacent to each other, either vertically or horizontally?

Solution 1
Call the different colors A,B,C. There are ways to rearrange these
colors to these three letters, so must be multiplied after the letters are permuted

in the grid. WLOG assume that A is in the center. In this


configuration, there are two cases, either all the A's lie on the same

diagonal: or all the other two A's are on adjacent


corners: In the first case there are two ways to order them since
there are two diagonals, and in the second case there are four ways to order
them since there are four pairs of adjacent corners.
In each case there is only one way to put the three B's and the three C's as

shown in the diagrams. This means that there

are ways to arrange A,B, and C in the grid, and there are 6 ways
to rearrange the colors. Therefore, there are ways in total, which

is .

-happykeeper

Solution 2 (Casework)
Without the loss of generality, we place a red ball in the top-left square. There are
two cases:
Case (1): The two balls adjacent to the top-left red ball have different

colors. Each placement has permutations, as there


are ways to permute RBG.
There are three sub-cases for Case
(1):

So, Case (1) has ways.

Case (2): The two balls adjacent to the top-left red ball have the same
color. Each placement has permutations, as there

are ways to choose three balls consisting of exactly two


colors (RBB, RGG, BRR, BGG, GRR, GBB). There are three sub-cases for Case
(2):

So, Case (2) has ways.

Together, the answer is


~MRENTHUSIASM

Solution 3 (Casework and Derangements)


Case (1): We have a permutation of R, B, and G as all of the rows. There
are ways to rearrange these three colors. After finishing the first row, we move
onto the second. Notice how the second row must be a derangement of the first

one. By the derangement formula, , so there are two possible


permutations of the second row. (Note: You could have also found the number of
derangements of PIE). Finally, there are possible permutations for the last row.
Thus, there are possibilities.

Case (2): All of the rows have two balls that are the same color and one that is
different. There are obviously possible configurations for the first row, for the
second, and for the third. .

Therefore, our answer is

You might also like